Đến nội dung

nguyenhiep1999 nội dung

Có 24 mục bởi nguyenhiep1999 (Tìm giới hạn từ 28-04-2020)


Sắp theo                Sắp xếp  

#639104 ABHM NỘI TIẾP

Đã gửi bởi nguyenhiep1999 on 09-06-2016 - 09:56 trong Hình học phẳng

HM là đường trung bình của tam giác CHB => HM vuông góc CD

=>$\angle MHA=\angle MCB$ 

để chứng minh ABHM nội tiếp ta chứng minh $\bigtriangleup BMC \infty \bigtriangleup AMH$

<=> CM.AH=BC.MH

mà theo định lí menelauyt cho tam giác ABH cát tuyến DJC ta có HJ=$\frac{1}{2}$AH

có $\frac{CM}{MH}=\frac{CH}{HJ}=\frac{BC}{AH}$ => dccm




#638321 CMR MN vuông góc với OI và bán kinh ngoại tiếp (AMN )=OI

Đã gửi bởi nguyenhiep1999 on 05-06-2016 - 16:58 trong Hình học phẳng

1.gọi T là trung điểm BN,J là trung điểm CN thì tâm thuộc đường tròn ngoại tiếp tam giác TJN.

2. MN vuông góc với OI thì dùng các nô

bán kính thì mình tính lằng nhằng lắm nhưng cuối cùng vẫn ra có cách nào nhanh ko?




#584680 chứng minh $a\overrightarrow{IA}+b\overrightarrow...

Đã gửi bởi nguyenhiep1999 on 24-08-2015 - 19:04 trong Hình học phẳng

đùa à bài này trong tài liệu chuyên có hỏi chi vậy




#555721 n số liên tiếp không là lũy thừa của một số nguyên tố

Đã gửi bởi nguyenhiep1999 on 22-04-2015 - 20:51 trong Số học

bài toán:chứng minh rằng với mọi  tự nhiên luôn tồn tai n só tự nhiên liên tiếp sao cho bất ký số nào trong số ấy đều không phải là lũy thừa của một số nguyên tố




#553711 chủ đề bị khóa

Đã gửi bởi nguyenhiep1999 on 13-04-2015 - 16:56 trong Xử lí vi phạm - Tranh chấp - Khiếu nại

tại sao em viết chủ đề nào cũng bị khóa vậy




#553690 Tập ước nguyên tố.

Đã gửi bởi nguyenhiep1999 on 13-04-2015 - 13:26 trong Số học

Bài 1. Cho $a \in \mathbb{Z},p$ là số nguyên tố thỏa mãn $(a,p)=1.$ Chứng minh rằng $a^p-1$ và $a-1$ không có cùng tập ước nguyên tố.

Bài 2. Cho $a,b \in \mathbb{N},p$ là số nguyên tố lẻ sao cho $(a,p)=(b,p)=1.$ Chứng minh $a^p-b^p$ và $a-b$ không có cùng tập ước nguyên tố.

Bài 3. Cho $a \in \mathbb{N},a>1.$ Tìm $n \in \mathbb{N}$ để $a^n-1$ và $a-1$ có cùng tập ước nguyên tố.

Bài 4. Tìm $a,m,n \in \mathbb{N}$ sao cho $(a-1)^m=a^n-1.$

Bài 5. Tìm $a,m,n \in \mathbb{N}$ sao cho $a^m-1$ và $a^n-1$ có cùng tập ước nguyên tố.

Bài 6. Cho $a,b \in \mathbb{N}.$ Tìm $n \in \mathbb{N}$ sao cho $a^n-b^n$ và $a-b$ có cùng tập ước nguyên tố.

Bài 7. Tìm $a,b,m,n \in \mathbb{N}$ sao cho $(a,b)=1$ và $a^m-b^m=(a-b)^n.$




#553496 bai toán 1

Đã gửi bởi nguyenhiep1999 on 12-04-2015 - 15:52 trong Bất đẳng thức và cực trị

$\sum \sqrt{\frac{a^{2}}{a^{2}+b+c}}\leq \sqrt{3}$  với mọi a,b,c>0,a2+b2+c2=3




#553488 tập ước nguyên tố

Đã gửi bởi nguyenhiep1999 on 12-04-2015 - 15:33 trong Số học

cho hai số nguyên a,b và số nguyên tố p sao cho:(a.p)=(b,p)=1.Chứng minh:ap-bp,a-b không có cùng tập ước nguyên tố.

chú thích:tập ước nguyên tố của số nguyem m là tập hợp tất cả các số nguyên tố là ước của m. :angry:




#553401 $\sum \frac{a^3}{2a^2+bc}\leq \f...

Đã gửi bởi nguyenhiep1999 on 12-04-2015 - 09:59 trong Bất đẳng thức - Cực trị

bài này xét hiệu đặt a^3,b^3,c^3 ra ngoài.trong sau khi phân tích nhân có chứa a2 +bc-b2-c2,b^2+ca-a^2-b^2,c^2+ab-a^2-b^2.

đặt ta có:

$\sum$ A.(a2 +bc-b2-c2) =A.(a^2+bc-b^2-c^2-b^2-ac+b^2+c^2)+(A+B).(b^2+ac-a^2-c^2-c^2-ab+a^2+b^2)+(A+B+C).(c^2+ab-a^2-b^2)

đánh giá a,b,c là xong.




#553380 $P_{M/O} = -(\dfrac{S_a.MA^2+S_b.MB^2+S_c.MC^2}...

Đã gửi bởi nguyenhiep1999 on 12-04-2015 - 09:24 trong Hình học phẳng

PM/O =OM2-R2 => nếu cho OM không đổi thì PM/O cũng không đổi nhưng điều này là vô ký => bài toán sai đấy bạn ạ.




#553374 $a^5+b^5+c^5 \geq a^4.b+b^4.c+c^4.a$

Đã gửi bởi nguyenhiep1999 on 12-04-2015 - 09:19 trong Bất đẳng thức và cực trị

thục ra bài này dung bất đẳng thức hoán vị cơ.




#553363 $a^5+b^5+c^5 \geq a^4.b+b^4.c+c^4.a$

Đã gửi bởi nguyenhiep1999 on 12-04-2015 - 08:58 trong Bất đẳng thức và cực trị

bài toán:cho a,b,c, $\alpha$ , $\beta$ > 0 .chứng minh rằng:

1/a5+b5+c5 $\geq$ a4.b+b4.c+c4.a

2/a^( $\alpha$ + $\beta$ ) +b^( $\alpha$ + $\beta$ ) +c^($\alpha$ + $\beta$ ) $\geq$ $\sum_{a}^{b}$ a^$\alpha$ . b^ $\beta$




#552562 Hỏi tồn tại hay không số tự nhiên n thỏa mãn $n^{2}+2^{n...

Đã gửi bởi nguyenhiep1999 on 08-04-2015 - 22:11 trong Số học

bạn ơi,theo đề bài $p=4k+1$ cơ mà sao có dạng $p=2$được.Với lại,bạn gõ Latex cho dễ nhìn.

nhầm.mình không biết gõ kiểu kia hay lỗi phông lắm.




#552552 phương trinh mũ là một biến.

Đã gửi bởi nguyenhiep1999 on 08-04-2015 - 22:00 trong Hình học

bài toán:tìm các số tự nhiên n,a sao cho an+1=na.

:closedeyes: bài này cần dùng đến lý thuyết cấp số nên bạn nào chưa oc không nên thử.




#552549 Hỏi tồn tại hay không số tự nhiên n thỏa mãn $n^{2}+2^{n...

Đã gửi bởi nguyenhiep1999 on 08-04-2015 - 21:51 trong Số học

bạn nêu cách giải của mình ra đi,mình kiến thức hạn hẹp nên còn suy nghĩ đây.Bạn không cần dùng $modun$ à?

từ đề bài dễ dàng suy ra n chẵn.

đặt n=2^q.k=>(2^q.k)2+(2^k)2^q chia hết cho p.

p=2.thỏa mãn.

p>2=>p lẻ.

=>k2+2k.2^q-2.q chia hết cho p.

bài toán quy về chứng minh :mọi ước nguyên tó lẻ của a2+b2 đều có dạng 4k+1trong đó (a;b)=1.

dễ thấy (a,p)=(b,p)=1

theo định lý fec-ma : ap-1-1 chia hết cho p,bp-1-1 chia hết cho p

=>ap-1-bp-1 chia hết cho p(1)

mà a2  đồng dư với b2 mod p =>ap+1-bp+1 chia hết cho p.

Nếu p=4k+3 thi p+1=4k+4 chia hết cho 4 =>ap+1  đồng dư với bp+1 mod p mà (1) =>bp+1  .2 chia hết cho p=>b chia hết cho p vô lí.

=>mọi ước nguyên tố của a2+b2 đều có dạng 4k+1.

đây hẳn là một bà khai thai ngược từ bài toán trên nhưng kết luận rằng không phải với bất kì số nguyên tố p nào cũng tồn tại n nguyên thỏa mãn yêu cầu đề bài.




#552485 Tông quát từ một bài toán.

Đã gửi bởi nguyenhiep1999 on 08-04-2015 - 19:58 trong Hình học phẳng

cho tam giác ABC.Chứng minh:ma+mb+m nhỏ hơn hoặc bằng 9\2R

Tông quát :cho đa giác lồi (A1A2...An) nội tiếp đường tròn bán kinh R, có trọng tâm G.Chứng minh rằng nRa lớn hơn hoặc bằng GA1a+...+GAn với mọi a>0.

để làm bai trên  cần chứng minh trong trường hợp a=1

sau đó sử dụng bất đẳng thúc bec-nu-li để chứng minh.




#552475 Hỏi tồn tại hay không số tự nhiên n thỏa mãn $n^{2}+2^{n...

Đã gửi bởi nguyenhiep1999 on 08-04-2015 - 19:37 trong Số học

mình cũng không chắc lắm

$p=4k+1\Rightarrow 2p=8k+2\equiv 2(mod 8)$

Lại có $n^{2}$ là số chính phương nên $n^{2}\equiv 1(mod 8)$                      (1)

Mà $2^{n}\equiv 1(mod 8)( n=0) ;2^{n}\equiv 2(mod 8) (n=1); 2^{n}\equiv 4(mod 8) (n=2); 2^{n}\equiv 0(mod 8)(n>2,n\epsilon N)$           (2)

Cộng (1) với (2) chỉ thấy n=0 là thoả mãn nhưng thay vào đẳng thức thì không đúng.

Vậy không tồn tại số tự nhiên n thoả m

cậu lập luận chẳng logic tí gì

sao tự nhiên lại cho n  đồng dư 1 mod 8,trong bài này thì n phải chãn chứ.




#552470 $\frac{IA^{2}}{c(p-a)}+\frac...

Đã gửi bởi nguyenhiep1999 on 08-04-2015 - 19:26 trong Hình học phẳng

Cho tam giác ABC có $a=BC,b=CA,c=AB$. Gọi I, p lần lượt là tâm đường tròn nội tiếp, nửa chu vi của tam giác ABC. Chứng minh rằng

$$\frac{IA^{2}}{c(p-a)}+\frac{IB^{2}}{a(p-b)}+\frac{IC^{2}}{b(p-c)}=2$$

bài này có vẻ chát đáy 

một hê thúc đẹp nếu đúng.

nếu cậu khẳng định nó đúng thì có phải bài này giải bàng cách tách IA2 thành r2+4.(p-a)2 và có r2=(p-a).(p-b).(p-c)/p

không?thử kiểm tra lại đề đi nhé xem nó có thật là đẳng thức hay không?




#552466 cho tam giác ABC cân tại A điểm M trên AB sao cho AB=3MA. H là hình chiếu của...

Đã gửi bởi nguyenhiep1999 on 08-04-2015 - 19:07 trong Hình học phẳng

cho tam giác ABC cân tại A điểm M trên AB sao cho AB=3MA. H là hình chiếu của B lên CM. I là trung điểm HC. Chứng minh rằng BI vuông góc IA

từ bài toán này:gọi Q là trung điểm của BC=>Tứ giác AIQC nội tiếp =>góc IQC=góc IAB =>MCB=ABI =>tam giác MBI và MCB đồng dạng.

rút ra 

1/MB.MB=MI.MC

2/góc MIB=góc ABC.




#552449 Hỏi tồn tại hay không số tự nhiên n thỏa mãn $n^{2}+2^{n...

Đã gửi bởi nguyenhiep1999 on 08-04-2015 - 18:00 trong Số học




#552439 1/ $x^2+3x-2=\frac{9x\sqrt{3x+1}}{(1+...

Đã gửi bởi nguyenhiep1999 on 08-04-2015 - 17:16 trong Phương trình, hệ phương trình và bất phương trình

bài này chắc là giải thế này:

chuyển 2x sang vế phải,đặt x ra ngoài rồi phân tích đa thức thành nhân tử.bài toán có nghiệm là 1 nên nhân liên hợp làm xuất hiện x-1 rồi chúng minh phần còn lại không âm là xong.




#552429 $M\in \Delta ABC$, $D\in AM, E \in BM, F...

Đã gửi bởi nguyenhiep1999 on 08-04-2015 - 16:43 trong Hình học phẳng

bài này dùng định lý xê-va sin hay hơn nhiều vì chỉ cần áp dụng liên tiếp định lý xê- va sin là ra chứ không phải biến đổi lằng nhằng gì cả.Thử dùng định lý Xê - va sin cho tam giác ABC với điểm P,Q,R rồi dung tiếp cho tam giác MBC với điểm P thì sẽ ra ngay.

thế cậu làm được bai hệ quả chưa. :icon6: ha




#552426 $M\in \Delta ABC$, $D\in AM, E \in BM, F...

Đã gửi bởi nguyenhiep1999 on 08-04-2015 - 16:41 trong Hình học phẳng

(a) Gọi $S$ là giao điểm của $EF$ với $BC$. Áp dụng Menelaus cho $\Delta MBC$ với $\overline{SEF}$, cho $\Delta BMD$ với $\overline{ERA}$ và cho $\Delta DMC$ với $\overline{FQA}$ suy ra $BC, EF, QR$ đồng quy.

Tương tự ta có $CA, PR, DF$ đồng quy và $AB, PQ, ED$ đồng quy.

Theo định lý Desargues ta có điều phải chứng minh.

(b) Áo dụng kết quả đầu tiên ở câu (a) cho ta $AX, BY, CZ$ đồng quy.

bài này dùng định lý xê-va sin hay hơn nhiều vì chỉ cần áp dụng liên tiếp định lý xê- va sin là ra chứ không phải biến đổi lằng nhằng gì cả.Thử dùng định lý Xê - va sin cho tam giác ABC với điểm P,Q,R rồi dung tiếp cho tam giác MBC với điểm P thì sẽ ra ngay.

thế cậu làm được bai hệ quả chưa. :icon6: hay nhỉ.




#552387 $M\in \Delta ABC$, $D\in AM, E \in BM, F...

Đã gửi bởi nguyenhiep1999 on 08-04-2015 - 13:35 trong Hình học phẳng

BÀI TOÁN ĐỒNG QUY HAY

 

Bài toán 1:cho tam giác ABC tùy ý.Lấy điểm M nằm trong tam giác.Trên AM,BM,CM lần lượt lấy các điểm D,E,F.Gọi P,Q,R lần lươt là giao điểm của BF và CE,AF và CD,AE và BD.Chứng minh AP,BQ,CR đồng quy.

b/gọi giao của MP,MQ,MR với 3 cạnh tam giác là X,Y,Z .chứng minh AX,BY,CZ đồng quy.

Hệ quả:cho tam giác ABC M,N,P lần lượt là trung điểm cua BC,CA,AB.O nằm trong tam giác.Gọi X,Y,Z lần lượt là trung điểm của OM,ON,OP.Chứng minh AX,BY,CZ đồng quy.

:mellow: Bài này sử dụng liên tiếp nhiều lần định lý Xê- va rất là hay và đẹp đó. :ohmy: